1answer.
Ask question
Login Signup
Ask question
All categories
  • English
  • Mathematics
  • Social Studies
  • Business
  • History
  • Health
  • Geography
  • Biology
  • Physics
  • Chemistry
  • Computers and Technology
  • Arts
  • World Languages
  • Spanish
  • French
  • German
  • Advanced Placement (AP)
  • SAT
  • Medicine
  • Law
  • Engineering
Mkey [24]
3 years ago
5

You plan to retire in 30 years and plan to contribute the same amount of money each year to your retirement fund. The fund earns

7% compounded annually. If you would like to withdraw $100,000 each year for 20 years, starting 1 year after the last contribution is made. Approximately how much money should you contribute to your retirement fund each year
Business
1 answer:
tatyana61 [14]3 years ago
3 0

Answer:

$11,215.24

Explanation:

After retirement:

Annual Withdrawal = $100,000

Period = 20 years

Annual Interest Rate = 7%

Amount required at retirement = $100,000 * PVIFA(7%, 20)

Amount required at retirement = $100,000 * (1 - (1/1.07)^20) / 0.07

Amount required at retirement = $100,000 * 10.5940

Amount required at retirement = $1,059,400

Before retirement:

Period = 30 years

Annual Deposit * FVIFA(7%, 30) = $1,059,400

Annual Deposit * (1.07^30 - 1) / 0.07 = $1,059,400

Annual Deposit * 94.46079 = $1,059,400

Annual Deposit = $11,215.24

So, you should contribute $11,215.24 each year into your retirement fund.

You might be interested in
Wax music expects sales of $437,500 next year. the profit margin is 4.8 percent, and the firm has a 30 percent dividend payout r
zimovet [89]

$16,231 is the Projected Increase in Retained Earnings.

<h3>Explanation</h3>

get here first Expected Profit that is express as

expected Profit = Sales × Profit Margin   .......................1

expected Profit = 437500 × 5.3%

expected Profit = $23187.50

and Dividends is here as

Dividends = Expected Profit × Dividend Payout Ratio   .................2

Dividends = 23187.50  × 30%

Dividends = $6956.25

Projected Increase in Retained Earnings will be

Projected Increase in Retained Earnings = expected Profit - Dividends   ........3

Projected Increase in Retained Earnings  = $23187.50 - $6956.25

Projected Increase in Retained Earnings = $16231.25

There are options missing in the question which is given below-

a. $16,231

b. $17,500

c. $18,300

d. $20,600

e. $21,000

Thus, the correct option is a. $16231

For more details about the question, click here:

brainly.com/question/14275701

#SPJ1

3 0
2 years ago
Consider a hypothetical economy in which the marginal propensity to consume (MPC) is 0.50. That is, if disposable income increas
evablogger [386]

Answer:

The level of saving =  $450 billion - $400 billion= $50 billion

Marginal propensity to save = 1- marginal propensity to consume (MPC)=0.5

Expected consumption

MPC=  change in Consumption/ change in income 200 billion * 0.5 = $100billion

Therefore consumption = 100 billion + 400 billion = $500 billion

Saving = $650 billion - $500 billion=  $ 150 billion

Explanation:

4 0
4 years ago
Bruce, a first-line supervisor at Lou’s Market, regularly uses abusive and derogatory words when dealing with his subordinates.
Burka [1]

Answer:

The correct answer is Interpersonal.

Explanation:

Interpersonal justice refers to the perception of justice of employees in the interpersonal treatment they receive from those who have the power and the power to distribute the results (usually managers and managers). It is important for managers to be courteous and polite and treat employees with dignity and respect to promote interpersonal justice. In addition, managers and managers must refrain from making derogatory comments or belittling their subordinates.

4 0
3 years ago
Customer Information Programs (CIPs) use several examples to define a person opening a new account. In which scenario should the
xz_007 [3.2K]

Answer:

Option (D) is the right answer.

Explanation:

According to the scenario, the most appropriate answer is option ( D) because Stanley smith is a customer of the bank as he has a checking account in the bank.

While the other options are wrong because of the following reasons:

  • Option (A) is incorrect because the company lucky licks Inc. is the customer of the bank, not any person.
  • Option (B) is incorrect because Cara is not the customer of the bank as she only uses the bank's ATM.
  • Option (C) is incorrect because Herman is not the customer of the bank as he only cashes his checks in the bank.
8 0
3 years ago
Alex doesn't care about where he lives, but he does care about what he eats. Alex spends all his money on restaurant meals at ei
Levart [38]

Answer:

Part 1). Yes, Alex be better off in Paris if the relative prices between American and French meals (p_a/p_f) are lower in Paris than Austin because his capability to buying goods growths so his cash is fundamentally value more locally.

Part 2) No, Alex be better off in Paris if the relative prices between American and French meals (p_a/p_f) are higher in Paris than Austin because even though he can buying the same components, they will cost extra, and he will consume less goods than living in Austin

5 0
3 years ago
Other questions:
  • Rather than offering its members ________ benefits, netroots associations often offer their members ________ benefits.
    9·1 answer
  • According to this excerpt, what is one major way in which the world bank differs from a regular bank?
    10·2 answers
  • Is it possible for a country to have a comparative advantage in producing a good without also having an absolute​ advantage? A c
    15·1 answer
  • Based on your understanding of the IS-LM model, graphically illustrate and explain what effect a reduction in consumer confidenc
    12·1 answer
  • 1.
    5·2 answers
  • A lease agreement that qualifies as a finance lease calls for annual lease payments of $50,000 over a four-year lease term (also
    11·1 answer
  • Negative transfer is said to have occurred when: a. A trainee dislikes the training sessions b. A trainee's performance declines
    12·2 answers
  • Your real estate agent mentions that homes in your price range require a payment of $1,200 per month for 30 years at 0.75% inter
    12·1 answer
  • The process by which ANZ bank's internal job descriptions are matched to external jobs with similar responsibilities to identify
    15·1 answer
  • 3. A U.S. MNC needs to raise capital of $100 million by issuing bonds. The firm can either raise US$ at 5% interest rate or issu
    10·1 answer
Add answer
Login
Not registered? Fast signup
Signup
Login Signup
Ask question!